Đến nội dung

Hình ảnh

$$\left|(-\it{1})^{\left\lfloor\frac{\it{k}}{\it{2}}\right\rfloor}\right|\geqq$$

* * * * * 1 Bình chọn n vars inequality inequalities combinatorics công thức waring (!) variables theorem variables lemma

  • Please log in to reply
Chưa có bài trả lời

#1
DOTOANNANG

DOTOANNANG

    Đại úy

  • ĐHV Toán Cao cấp
  • 1609 Bài viết

Với $\text{V}_{\,\it{m}}\left ( \it{n} \right )= \frac{\it{1}}{\binom{\it{n}}{\it{m}}}\sum\limits_{\it{1}\leqq \it{i}_{\,\it{1}}< \it{i}_{\,\it{2}}< \it{i}_{\,\it{3}}< \,...\,\it{i}_{\,\it{m}}\leqq \it{n}} \left ( \prod\limits_{\it{j}= \it{0}}^{\,\it{m}} \it{x}_{\,\it{i}\left ( \it{j} \right )} \right ),\,\it{x}_{\,\it{i}\left ( \it{0} \right )}= \it{1},\,\text{V}_{\,\it{0}}\left ( \it{n} \right )= \it{1},\,\it{x}_{\,\,\overline{\it{j},\,\it{m}}}> \it{0}$ . Chứng minh rằng với bất kì số  $\it{k}\geqq \it{m}$  thì:

$\left | \left ( -\,\it{1} \right )^{\left \lfloor \frac{\it{k}}{\it{2}} \right \rfloor}\sum\limits_{\it{r}= \it{m}}^{\,\it{k}}\left ( -\,\it{1} \right )^{\,\it{r}}\binom{\it{k}- \it{1}}{\it{r}}\,\text{V}_{\,\it{r}}\text{V}_{\,\it{k}- \it{r}} \right |\geqq \left | \left ( -\,\it{1} \right )^{\left \lfloor \frac{\it{j}}{\it{2}} \right \rfloor}\sum\limits_{\it{j}}\left ( -\,\it{1} \right )^{\,\it{j}}\binom{\it{m}- \it{1}}{\it{j}}\,\text{V}_{\,\it{j}}\text{V}_{\,\it{m}- \it{j}} \right |$







Được gắn nhãn với một hoặc nhiều trong số những từ khóa sau: n vars, inequality, inequalities, combinatorics, công thức waring (!), variables theorem, variables lemma

0 người đang xem chủ đề

0 thành viên, 0 khách, 0 thành viên ẩn danh